Last visit was: 24 Jan 2025, 16:15 It is currently 24 Jan 2025, 16:15
Close
GMAT Club Daily Prep
Thank you for using the timer - this advanced tool can estimate your performance and suggest more practice questions. We have subscribed you to Daily Prep Questions via email.

Customized
for You

we will pick new questions that match your level based on your Timer History

Track
Your Progress

every week, we’ll send you an estimated GMAT score based on your performance

Practice
Pays

we will pick new questions that match your level based on your Timer History
Not interested in getting valuable practice questions and articles delivered to your email? No problem, unsubscribe here.
Close
Request Expert Reply
Confirm Cancel
605-655 Level|   Assumption|               
User avatar
MBA2ran
Joined: 10 Jan 2006
Last visit: 14 Jun 2007
Posts: 64
Own Kudos:
747
 [237]
Posts: 64
Kudos: 747
 [237]
19
Kudos
Add Kudos
218
Bookmarks
Bookmark this Post
Most Helpful Reply
User avatar
KarishmaB
Joined: 16 Oct 2010
Last visit: 23 Jan 2025
Posts: 15,683
Own Kudos:
71,249
 [44]
Given Kudos: 452
Location: Pune, India
Expert reply
Active GMAT Club Expert! Tag them with @ followed by their username for a faster response.
Posts: 15,683
Kudos: 71,249
 [44]
30
Kudos
Add Kudos
14
Bookmarks
Bookmark this Post
User avatar
ScottTargetTestPrep
User avatar
Target Test Prep Representative
Joined: 14 Oct 2015
Last visit: 24 Jan 2025
Posts: 20,081
Own Kudos:
24,852
 [22]
Given Kudos: 289
Status:Founder & CEO
Affiliations: Target Test Prep
Location: United States (CA)
Expert reply
Active GMAT Club Expert! Tag them with @ followed by their username for a faster response.
Posts: 20,081
Kudos: 24,852
 [22]
18
Kudos
Add Kudos
4
Bookmarks
Bookmark this Post
General Discussion
User avatar
ankit0411
User avatar
BSchool Moderator
Joined: 28 May 2012
Last visit: 13 Oct 2014
Posts: 83
Own Kudos:
457
 [1]
Given Kudos: 11
Location: India
Concentration: General Management, Strategy
GPA: 3.33
WE:Information Technology (Retail: E-commerce)
1
Kudos
Add Kudos
Bookmarks
Bookmark this Post
carcass
The program to control the entry of illegal drugs into the country was a failure in 1987. If the program had been successful, the wholesale price of most illegal drugs would not have dropped substantially in 1987.

The argument in the passage depends on which of the following assumptions?


1) The supply of illegal drugs dropped substantially in 1987.

2)The price paid for most illegal drugs by the average consumer did not drop substantially in 1987.

3)Domestic production of illegal drugs increased at a higher rate than did the entry of such drugs into the country.

4)The wholesale price of a few illegal drugs increased substantially in 1987.

5)A drop in demand for most illegal drugs in 1987 was not the sole cause of the drop in their wholesale price.

Guys try this one. OA later . Thanks for discussion in advance :)

Let me just try -

1) The program to control the entry of illegal drugs into the country was a failure in 1987 - The program was a failure

2) If the program had been successful, the wholesale price of most illegal drugs would not have dropped substantially in 1987.

Thus, it means that the program was not successful ( there was illegal entry of drugs into the country) and hence, the prices dropped.

If I negate E - A drop in demand for most illegal drugs in 1987 was THE sole cause of the drop in their wholesale price.

okay - so if the price decrease was due to the drop in demand; the illegal entry of drugs was not the reason of their wholesale price decrease.

Am I right ?
User avatar
souvik101990
Joined: 19 Mar 2012
Last visit: 17 Jan 2025
Posts: 4,325
Own Kudos:
52,308
 [3]
Given Kudos: 2,326
Location: United States (WA)
Concentration: Leadership, General Management
GMAT 1: 760 Q50 V42
GMAT 2: 740 Q49 V42 (Online)
GMAT 3: 760 Q50 V42 (Online)
GPA: 3.8
WE:Marketing (Non-Profit and Government)
Products:
Expert reply
GMAT 3: 760 Q50 V42 (Online)
Posts: 4,325
Kudos: 52,308
 [3]
3
Kudos
Add Kudos
Bookmarks
Bookmark this Post
soniedarshan
The program to control the entry of illegal drugs into the country was a failure in
1987. If the program had been successful, the wholesale price of most illegal drugs
would not have dropped substantially in 1987.

The argument in the passage depends on which of the following
assumptions?

(A) The supply of illegal drugs dropped substantially in 1987.
(B) The price paid for most illegal drugs by the average consumer did not drop
substantially in 1987.
(C) Domestic production of illegal drugs increased at a higher rate than did the entry
of such drugs into the country.
(D) The wholesale price of a few illegal drugs increased substantially in 1987.
(E) A drop in demand for most illegal drugs in 1987 was not the sole cause of the drop
in their wholesale price.

E is correct. If you negate E, which becomes "Drop in demand WAS the sole cause of drop of wholesale price", the argument explodes into smithereens.
User avatar
smmunna
Joined: 09 Jan 2018
Last visit: 22 Apr 2019
Posts: 12
Own Kudos:
19
 [2]
Given Kudos: 1,059
Posts: 12
Kudos: 19
 [2]
2
Kudos
Add Kudos
Bookmarks
Bookmark this Post
MBA2ran
The program to control the entry of illegal drugs into the country was a failure in 1987. If the program had been successful, the wholesale price of most illegal drugs would not have dropped substantially in 1987.

The argument in the passage depends on which of the following assumptions?

(A) The supply of illegal drugs dropped substantially in 1987.
(B) The price paid for most illegal drugs by the average consumer did not drop substantially in 1987.
(C) Domestic production of illegal drugs increased at a higher rate than did the entry of such drugs into the country.
(D) The wholesale price of a few illegal drugs increased substantially in 1987.
(E) A drop in demand for most illegal drugs in 1987 was not the sole cause of the drop in their wholesale price.


The only choice that must be true in order to conclude legitimately from the drop in wholesale price of illegal drugs that the program was a failure is choice E, the best answer. If the drop in price was caused by a drop in demand, there is no reason to suspect that there has been any increase in supply caused by drugs entering the country.

The other choices can be false without affecting the argument. The supply of illegal drugs need not have dropped (choice A), and the retail price could have dropped (choice B). The entry of illegal drugs could have risen at a higher rate than domestic production (choice C), and no illegal drug need have undergone a substantial price rise (choice D).
User avatar
Shrey08
Joined: 04 Mar 2020
Last visit: 06 Apr 2024
Posts: 129
Own Kudos:
Given Kudos: 304
Location: India
GMAT 1: 640 Q47 V30
GMAT 1: 640 Q47 V30
Posts: 129
Kudos: 143
Kudos
Add Kudos
Bookmarks
Bookmark this Post
MBA2ran
The program to control the entry of illegal drugs into the country was a failure in 1987. If the program had been successful, the wholesale price of most illegal drugs would not have dropped substantially in 1987.

The argument in the passage depends on which of the following assumptions?


(E) A drop in demand for most illegal drugs in 1987 was not the sole cause of the drop in their wholesale price.


Hi VeritasKarishma

I chose 'E', however, I was a little skeptical about the verbiage. Don't you think the usage "...was not the solecause of the drop..." is extreme ? I mean, author doesn't have to assume that demand was not the "sole" cause of the drop. It could have been one of the 2 causes, Right ?
User avatar
KarishmaB
Joined: 16 Oct 2010
Last visit: 23 Jan 2025
Posts: 15,683
Own Kudos:
71,249
 [1]
Given Kudos: 452
Location: Pune, India
Expert reply
Active GMAT Club Expert! Tag them with @ followed by their username for a faster response.
Posts: 15,683
Kudos: 71,249
 [1]
Kudos
Add Kudos
1
Bookmarks
Bookmark this Post
ShreyKapil08
MBA2ran
The program to control the entry of illegal drugs into the country was a failure in 1987. If the program had been successful, the wholesale price of most illegal drugs would not have dropped substantially in 1987.

The argument in the passage depends on which of the following assumptions?


(E) A drop in demand for most illegal drugs in 1987 was not the sole cause of the drop in their wholesale price.


Hi VeritasKarishma

I chose 'E', however, I was a little skeptical about the verbiage. Don't you think the usage "...was not the solecause of the drop..." is extreme ? I mean, author doesn't have to assume that demand was not the "sole" cause of the drop. It could have been one of the 2 causes, Right ?

Option (E) makes sense because it uses the word "sole". I don't know why many test takers think in terms of "the option is extreme" but it is not valid. There are no extreme options. In some cases, words such as "all", "none" etc are justified and in others, they are not - just like any other word. You need to see whether it is justified in the case presented to you.

When the author says that price drop in 1987 implies that supplies continued at a healthy level or even increased, he is assuming that the reason for the price drop is related to the supply side. This means he is assuming that fall in demand is not the sole reason.
User avatar
Hovkial
Joined: 23 Apr 2019
Last visit: 24 Nov 2022
Posts: 805
Own Kudos:
2,121
 [1]
Given Kudos: 203
Status:PhD trained. Education research, management.
Posts: 805
Kudos: 2,121
 [1]
Kudos
Add Kudos
1
Bookmarks
Bookmark this Post
OFFICIAL GMAT EXPLANATION

The only choice that must be true in order to conclude legitimately from the drop in wholesale price of illegal drugs that the program was a failure is choice E, the best answer. If the drop in price was caused by a drop in demand, there is no reason to suspect that there has been any increase in supply caused by drugs entering the country.

The other choices can be false without affecting the argument. The supply of illegal drugs need not have dropped (choice A), and the retail price could have dropped (choice B). The entry of illegal drugs could have risen at a higher rate than domestic production (choice C), and no illegal drug need have undergone a substantial price rise (choice D).
User avatar
ashmit99
Joined: 20 Feb 2019
Last visit: 24 Jan 2025
Posts: 82
Own Kudos:
Given Kudos: 189
Location: India
GPA: 3.2
Products:
Posts: 82
Kudos: 34
Kudos
Add Kudos
Bookmarks
Bookmark this Post
MBA2ran
The program to control the entry of illegal drugs into the country was a failure in 1987. If the program had been successful, the wholesale price of most illegal drugs would not have dropped substantially in 1987.

The argument in the passage depends on which of the following assumptions?


(A) The supply of illegal drugs dropped substantially in 1987.

(B) The price paid for most illegal drugs by the average consumer did not drop substantially in 1987.

(C) Domestic production of illegal drugs increased at a higher rate than did the entry of such drugs into the country.

(D) The wholesale price of a few illegal drugs increased substantially in 1987.

(E) A drop in demand for most illegal drugs in 1987 was not the sole cause of the drop in their wholesale price.

Related question (weaken): https://gmatclub.com/forum/the-program-to-control-the-entry-of-illegal-drugs-into-the-weaken-137754.html

The only choice that must be true in order to conclude legitimately from the drop in wholesale price of illegal drugs that the program was a failure is choice E, the best answer. If the drop in price was caused by a drop in demand, there is no reason to suspect that there has been any increase in supply caused by drugs entering the country.

The other choices can be false without affecting the argument. The supply of illegal drugs need not have dropped (choice A), and the retail price could have dropped (choice B). The entry of illegal drugs could have risen at a higher rate than domestic production (choice C), and no illegal drug need have undergone a substantial price rise (choice D).


Hello experts,

C'. Domestic production of illegal drugs DID NOT increase at a higher rate than did the entry of such drugs into the country.

Can this be an assumption too?

VeritasKarishma ScottTargetTestPrep AndrewN
avatar
AndrewN
avatar
Volunteer Expert
Joined: 16 May 2019
Last visit: 24 Jan 2025
Posts: 3,503
Own Kudos:
Given Kudos: 500
Expert reply
Posts: 3,503
Kudos: 7,144
Kudos
Add Kudos
Bookmarks
Bookmark this Post
ashmit99
The argument in the passage depends on which of the following assumptions?
Hello experts,

C'. Domestic production of illegal drugs DID NOT increase at a higher rate than did the entry of such drugs into the country.

Can this be an assumption too?

VeritasKarishma ScottTargetTestPrep AndrewN
Hello, ashmit99. You have to read the question stem carefully to avoid associative reasoning that can lead you wayward. We are not interested in assumptions that could be made, but in the one assumption of the options provided that the argument depends on. In other words, if such an answer were untrue, then the argument as presented would collapse. I would hope that the earlier responses in the thread help clear any doubts about (C), but if you still have questions, let me know.

- Andrew
User avatar
ashmit99
Joined: 20 Feb 2019
Last visit: 24 Jan 2025
Posts: 82
Own Kudos:
Given Kudos: 189
Location: India
GPA: 3.2
Products:
Posts: 82
Kudos: 34
Kudos
Add Kudos
Bookmarks
Bookmark this Post
AndrewN
ashmit99
The argument in the passage depends on which of the following assumptions?
Hello experts,

C'. Domestic production of illegal drugs DID NOT increase at a higher rate than did the entry of such drugs into the country.

Can this be an assumption too?

VeritasKarishma ScottTargetTestPrep AndrewN
Hello, ashmit99. You have to read the question stem carefully to avoid associative reasoning that can lead you wayward. We are not interested in assumptions that could be made, but in the one assumption of the options provided that the argument depends on. In other words, if such an answer were untrue, then the argument as presented would collapse. I would hope that the earlier responses in the thread help clear any doubts about (C), but if you still have questions, let me know.

- Andrew

Thanks for replying AndrewN

Got your point. But if I negate C' it would become the original option C.

So C says that domestic production increased at higher rate than did entry of such drugs into the country.

So basically now it seems like even if the program had been successful, the wholesale price still would have dropped substantially.

So in this case, the conclusion breaks.

Please help me with my doubt.

Thanks!
avatar
AndrewN
avatar
Volunteer Expert
Joined: 16 May 2019
Last visit: 24 Jan 2025
Posts: 3,503
Own Kudos:
7,144
 [3]
Given Kudos: 500
Expert reply
Posts: 3,503
Kudos: 7,144
 [3]
2
Kudos
Add Kudos
1
Bookmarks
Bookmark this Post
ashmit99
Thanks for replying AndrewN

Got your point. But if I negate C' it would become the original option C.

So C says that domestic production increased at higher rate than did entry of such drugs into the country.

So basically now it seems like even if the program had been successful, the wholesale price still would have dropped substantially.

So in this case, the conclusion breaks.

Please help me with my doubt.

Thanks!
Hello again, ashmit99. I did not say anything about negation exactly. Your goal is simply to find the one answer choice that necessarily bridges the gap between premise and conclusion. I use a drag-and-drop (or missing link) strategy instead of the negation technique when taking on such questions. Test (C) as is between the premise and conclusion:

Premise: If the program had been successful, the wholesale price of most illegal drugs would not have dropped substantially in 1987.

(C) Domestic production of illegal drugs increased at a higher rate than did the entry of such drugs into the country.

Conclusion: The program to control the entry of illegal drugs into the country was a failure in 1987.

Analysis: Why are we focused on a rate of growth between the domestic production of drugs and the importation of drugs? Whether the domestic production outpaced the importation is beside the point. The conclusion about the program to control the entry of illegal drugs into the country could hold, regardless of what may be going on with domestic production. Answer choice (C) cannot be our necessary assumption.

Repeat the process for (E):

Premise: If the program had been successful, the wholesale price of most illegal drugs would not have dropped substantially in 1987.

(E) A drop in demand for most illegal drugs in 1987 was not the sole cause of the drop in their wholesale price.

Conclusion: The program to control the entry of illegal drugs into the country was a failure in 1987.

Now this makes more sense. As evidence of the assertion, the argument points to the wholesale price of most illegal drugs. Choice (E) touches directly on this concern and informs us that demand for such drugs was not the sole cause of the drop in wholesale price, so there must have been other factors at work. Perhaps, then, the conclusion can be made that the program was a failure. At least the connection is clear, and that is all we are looking for in this sort of question.

I hope that makes more sense.

- Andrew
User avatar
KarishmaB
Joined: 16 Oct 2010
Last visit: 23 Jan 2025
Posts: 15,683
Own Kudos:
71,249
 [1]
Given Kudos: 452
Location: Pune, India
Expert reply
Active GMAT Club Expert! Tag them with @ followed by their username for a faster response.
Posts: 15,683
Kudos: 71,249
 [1]
1
Kudos
Add Kudos
Bookmarks
Bookmark this Post
ashmit99
MBA2ran
The program to control the entry of illegal drugs into the country was a failure in 1987. If the program had been successful, the wholesale price of most illegal drugs would not have dropped substantially in 1987.

The argument in the passage depends on which of the following assumptions?


(A) The supply of illegal drugs dropped substantially in 1987.

(B) The price paid for most illegal drugs by the average consumer did not drop substantially in 1987.

(C) Domestic production of illegal drugs increased at a higher rate than did the entry of such drugs into the country.

(D) The wholesale price of a few illegal drugs increased substantially in 1987.

(E) A drop in demand for most illegal drugs in 1987 was not the sole cause of the drop in their wholesale price.

Related question (weaken): https://gmatclub.com/forum/the-program-to-control-the-entry-of-illegal-drugs-into-the-weaken-137754.html

The only choice that must be true in order to conclude legitimately from the drop in wholesale price of illegal drugs that the program was a failure is choice E, the best answer. If the drop in price was caused by a drop in demand, there is no reason to suspect that there has been any increase in supply caused by drugs entering the country.

The other choices can be false without affecting the argument. The supply of illegal drugs need not have dropped (choice A), and the retail price could have dropped (choice B). The entry of illegal drugs could have risen at a higher rate than domestic production (choice C), and no illegal drug need have undergone a substantial price rise (choice D).


Hello experts,

C'. Domestic production of illegal drugs DID NOT increase at a higher rate than did the entry of such drugs into the country.

Can this be an assumption too?

VeritasKarishma ScottTargetTestPrep AndrewN

An option would need to be worded very carefully to make it an assumption.

Other than option (E), an assumption is: "Dramatic increase in domestic production was not the sole cause of price drop."

Rate of increase in domestic production vs rate of increase in illegal import doesn't make much sense.
avatar
Niha01
Joined: 22 Mar 2020
Last visit: 19 Dec 2022
Posts: 26
Own Kudos:
Given Kudos: 71
Posts: 26
Kudos: 1
Kudos
Add Kudos
Bookmarks
Bookmark this Post
AndrewN
ashmit99
Thanks for replying AndrewN

Got your point. But if I negate C' it would become the original option C.

So C says that domestic production increased at higher rate than did entry of such drugs into the country.

So basically now it seems like even if the program had been successful, the wholesale price still would have dropped substantially.

So in this case, the conclusion breaks.

Please help me with my doubt.

Thanks!
Hello again, ashmit99. I did not say anything about negation exactly. Your goal is simply to find the one answer choice that necessarily bridges the gap between premise and conclusion. I use a drag-and-drop (or missing link) strategy instead of the negation technique when taking on such questions. Test (C) as is between the premise and conclusion:

Premise: If the program had been successful, the wholesale price of most illegal drugs would not have dropped substantially in 1987.

(C) Domestic production of illegal drugs increased at a higher rate than did the entry of such drugs into the country.

Conclusion: The program to control the entry of illegal drugs into the country was a failure in 1987.

Analysis: Why are we focused on a rate of growth between the domestic production of drugs and the importation of drugs? Whether the domestic production outpaced the importation is beside the point. The conclusion about the program to control the entry of illegal drugs into the country could hold, regardless of what may be going on with domestic production. Answer choice (C) cannot be our necessary assumption.

Repeat the process for (E):

Premise: If the program had been successful, the wholesale price of most illegal drugs would not have dropped substantially in 1987.

(E) A drop in demand for most illegal drugs in 1987 was not the sole cause of the drop in their wholesale price.

Conclusion: The program to control the entry of illegal drugs into the country was a failure in 1987.

Now this makes more sense. As evidence of the assertion, the argument points to the wholesale price of most illegal drugs. Choice (E) touches directly on this concern and informs us that demand for such drugs was not the sole cause of the drop in wholesale price, so there must have been other factors at work. Perhaps, then, the conclusion can be made that the program was a failure. At least the connection is clear, and that is all we are looking for in this sort of question.

I hope that makes more sense.

- Andrew

Hi AndrewN

I was confused between B and E and chose B. My reason was-
If the average customer refused to pay high price for illegal drugs, the wholesale price of the commodity would have to be dropped. Where did I faulter in my reasoning here.
avatar
AndrewN
avatar
Volunteer Expert
Joined: 16 May 2019
Last visit: 24 Jan 2025
Posts: 3,503
Own Kudos:
Given Kudos: 500
Expert reply
Posts: 3,503
Kudos: 7,144
Kudos
Add Kudos
Bookmarks
Bookmark this Post
Niha01
Hi AndrewN

I was confused between B and E and chose B. My reason was-
If the average customer refused to pay high price for illegal drugs, the wholesale price of the commodity would have to be dropped. Where did I faulter in my reasoning here.
Hello, Niha01. In light of my earlier post, how about we test answer choice (B) as the assumption?

Claim: The program to control the entry of illegal drugs into the country was a failure in 1987.

Assumption: The price paid for most illegal drugs by the average consumer did not drop substantially in 1987.

Conditional: If the program had been successful, the wholesale price of most illegal drugs would not have dropped substantially in 1987.

I think a major problem with answer choice (B) as the supposed assumption is that the evidence in the conditional is based on the wholesale price, and wholesale in this context means the bulk price that, presumably, large-scale drug dealers would pay for shipments of drugs. Such dealers would then turn around and sell to others, and on down the chain, to the point of purchase between small-scale drug dealer and the average consumer. In other words, answer choice (B) is an after-the-fact consideration that has nothing to do with the wholesale price.

Perhaps the question makes more sense now. Thank you for thinking to follow up.

- Andrew
avatar
Niha01
Joined: 22 Mar 2020
Last visit: 19 Dec 2022
Posts: 26
Own Kudos:
Given Kudos: 71
Posts: 26
Kudos: 1
Kudos
Add Kudos
Bookmarks
Bookmark this Post
AndrewN
Niha01
Hi AndrewN

I was confused between B and E and chose B. My reason was-
If the average customer refused to pay high price for illegal drugs, the wholesale price of the commodity would have to be dropped. Where did I faulter in my reasoning here.
Hello, Niha01. In light of my earlier post, how about we test answer choice (B) as the assumption?

Claim: The program to control the entry of illegal drugs into the country was a failure in 1987.

Assumption: The price paid for most illegal drugs by the average consumer did not drop substantially in 1987.

Conditional: If the program had been successful, the wholesale price of most illegal drugs would not have dropped substantially in 1987.

I think a major problem with answer choice (B) as the supposed assumption is that the evidence in the conditional is based on the wholesale price, and wholesale in this context means the bulk price that, presumably, large-scale drug dealers would pay for shipments of drugs. Such dealers would then turn around and sell to others, and on down the chain, to the point of purchase between small-scale drug dealer and the average consumer. In other words, answer choice (B) is an after-the-fact consideration that has nothing to do with the wholesale price.

Perhaps the question makes more sense now. Thank you for thinking to follow up.

- Andrew

Thank you AndrewN

I backtracked average consumer price to wholesale price and assumed that it would get reduced too eventually.
avatar
AndrewN
avatar
Volunteer Expert
Joined: 16 May 2019
Last visit: 24 Jan 2025
Posts: 3,503
Own Kudos:
Given Kudos: 500
Expert reply
Posts: 3,503
Kudos: 7,144
Kudos
Add Kudos
Bookmarks
Bookmark this Post
Niha01
Thank you AndrewN

I backtracked average consumer price to wholesale price and assumed that it would get reduced too eventually.
Glad to help. You stated it just right in your response: you assumed something. In an assumption question in CR, you should not have to assume a sequence of events that is not clearly outlined in the passage. Rather, you want to find an answer choice that best fits the context described by the stated facts or claims.

Thank you for following up, and good luck with your studies.

- Andrew
User avatar
stackskillz
Joined: 28 Feb 2022
Last visit: 21 Jan 2025
Posts: 61
Own Kudos:
Given Kudos: 163
Posts: 61
Kudos: 12
Kudos
Add Kudos
Bookmarks
Bookmark this Post
This it the solution I came up with:

Conc: ­The program to control the entry of illegal drugs into the country was a failure in 1987.

This is a nice cause-effect passage. In short it states - Failure of program => Substantial drop in wholesale price of most illegal drugs

(A) The supply of illegal drugs dropped substantially in 1987. - This attacks the facts in the passage by saying the supply of illegal drugs dropped substantially, which woul have led to a price increase, rather than a price decrease stated. Drop

(B) The price paid for most illegal drugs by the average consumer did not drop substantially in 1987. - Does this mean the retailers were making money? The conclusion talks about wholesale price, but the option talks about retail/consumer price. Doesn't allow us to confirm whether the program was a failure or not. Drop

(C) Domestic production of illegal drugs increased at a higher rate than did the entry of such drugs into the country - So basically supply increased a lot and the program didn't fail to restrict the import of illegal drugs. This weakens the passage. Drop

(D) The wholesale price of a few illegal drugs increased substantially in 1987 - This can defintely co-exist with the reasoning of the passage, i.e., "Only a few drugs had a substantial increase in the wholesale price when all other illegal drugs dropped in price substantially". Drop

(E) A drop in demand for most illegal drugs in 1987 was not the sole cause of the drop in their wholesale price. - Looks good. Let's negate - "A drop in demand for most illegal drugs ... was the sole cause for the drop .. ". If this is true, then the program was not a failure. The drop was due to a different factor, i.e., cause-effect. Bingo. Keep­
User avatar
user1937
Joined: 04 Apr 2024
Last visit: 05 Jan 2025
Posts: 70
Own Kudos:
Given Kudos: 24
Posts: 70
Kudos: 29
Kudos
Add Kudos
Bookmarks
Bookmark this Post
ScottTargetTestPrep
MBA2ran
The program to control the entry of illegal drugs into the country was a failure in 1987. If the program had been successful, the wholesale price of most illegal drugs would not have dropped substantially in 1987.

13. The argument in the passage depends on which of the following assumptions?

(A) The supply of illegal drugs dropped substantially in 1987.
(B) The price paid for most illegal drugs by the average consumer did not drop substantially in 1987.
(C) Domestic production of illegal drugs increased at a higher rate than did the entry of such drugs into the country.
(D) The wholesale price of a few illegal drugs increased substantially in 1987.
(E) A drop in demand for most illegal drugs in 1987 was not the sole cause of the drop in their wholesale price.
While, generally, answering Critical Reasoning questions does not require the use of any sophisticated knowledge, understanding the logic of some Critical Reasoning questions requires a basic understanding of the economic laws of supply and demand. This question is such a question.

For anyone who is not familiar with the laws of supply and demand, here is a basic summary:

If the supply of a good increases and all other factors are held equal, normally the price of the good decreases. Conversely, if the supply of a good decreases, normally the price increases.

Demand affects price in the opposite way. If the demand for a good increases and all other factors are held equal, normally the price of the good increases. If demand decreases, normally the price decreases.

Now, let’s analyze the argument in this question.

The argument mentions a “program to control the entry of illegal drugs into the country.” This program is related to the laws of supply and demand in that the purpose of the program is to reduce the supply of illegal drugs in the country by controlling the entry of the drugs.

As discussed above, a reduction in supply should result in an increase in price, but the argument indicates that “the wholesale prices of most illegal drugs… dropped substantially.” In other words, even though there was a program meant to reduce supply, the prices fell.

Since one would expect that a reduction in supply would result in price increases rather than price decreases, the author of the argument reasons that since the prices decreased, supply must not have decreased, and therefore, the argument concludes, the program must have failed.

Now, let’s go to the answer choices to see which the author assumed in coming to that conclusion.

(A) The supply of illegal drugs dropped substantially in 1987.

Since the conclusion of the argument is that the program to reduce supply failed, this answer choice is not an assumption required for coming to that conclusion but rather is a statement in opposition to the conclusion.

(B) The price paid for most illegal drugs by the average consumer did not drop substantially in 1987.

This answer choice is incorrect in two ways.

The first is that the author is not concerned with the prices consumers pay, i.e., the retail prices of illegal drugs. The author is concerned with wholesale prices of illegal drugs and has already made the case that supply did not drop by pointing to the drop in wholesale prices.

The second is that the conclusion states that the program to reduce supply failed. Any drop in prices of illegal drugs would be consistent with that conclusion. Therefore, clearly, the argument does not rely on the assumption that the retail prices of illegal drugs did not drop.

(C) Domestic production of illegal drugs increased at a higher rate than did the entry of such drugs into the country.

This answer choice could be tempting because an increase in domestic production of illegal drugs would result in an increase in supply, even if there were a program in place to reduce supply from foreign sources by controlling entry of drugs into the country.

So, this answer choice could explain why, even though there was a program to reduce supply, the prices of illegal drugs decreased; supply from domestic sources could have offset any reduction in supply achieved by the program to limit entry.

However, we are not looking for an explanation. We are looking for an assumption. So, this choice is not our answer.

(D) The wholesale price of a few illegal drugs increased substantially in 1987.

The conclusion of the argument is that a program to reduce supply failed. Since a reduction in supply would push prices upward, price increases would indicate that the program to reduce supply had not failed but instead had succeeded. So, the conclusion does not depend on assuming that any prices of illegal drugs increased.

(E) A drop in demand for most illegal drugs in 1987 was not the sole cause of the drop in their wholesale price.

Correct. The argument’s conclusion, that the program to reduce supply failed, is based on the fact that the prices of illegal drugs dropped. In arriving at its conclusion, the argument relies on the assumption that there was not some other reason for the decreases in the prices of illegal drugs. Therefore, that a drop in demand was not the sole cause of the price drop is an assumption upon which the argument depends.

The correct answer is E.
 
­
Hi Scott. Thanks for the detailed answer. BUT almost all the time we hear advice not to use some knowledge that would be true in real world in the GMAT, and especially in assumption questions where assumption is accepted as true irrespective of their truth in real world. So, I was wondering if such a rule might hurt in answering the CR questions. Thanks.
 1   2   
Moderators:
GMAT Club Verbal Expert
7216 posts
GMAT Club Verbal Expert
234 posts